Вы находитесь на странице: 1из 6

A FRAMEWORK TO PRETHINK IN QUANTITATIVE PREDICTIVE ARGUMENTS Analysis of more than 300 OG questions suggests that almost 8-10% of the

CR questions belong to one category Quantitative Predictive Arguments - implying that in all likelihood, youll encounter one or two questions of this type in the actual GMAT. The purpose of this article is to help you build an efficient thought process for solving these questions and thereby, simultaneously improve accuracy and efficiency. To begin with, lets look at one OG question of this type. Take 2-3 minutes to solve this question. Near Chicago a newly built hydroponic spinach factory, a completely controlled environment for growing spinach, produces on 1 acre of floor space what it takes 100 acres of fields to produce. Expenses, especially for electricity, are high, however, and the spinach produced costs about four times as much as washed California field spinach, the spinach commonly sold throughout the United States. Which of the following, if true, best supports a projection that the spinach-growing facility near Chicago will be profitable? A. Once the operators of the facility are experienced, they will be able to cut operating expenses by 25 percent. B. There is virtually no scope for any further reduction in the cost per pound for California field spinach. C. Unlike washed field spinach, the hydroponically grown spinach is untainted by any pesticides or herbicides and thus will sell at exceptionally high prices to such customers as health food restaurants. D. Since spinach is a crop that ships relatively well, the market for the hydroponically grown spinach is no more limited to the Chicago area than the market for California field spinach is to California. E. A second hydroponic facility is being built in Canada, taking advantage of inexpensive electricity and high vegetable prices. After solving the question, think about the parameter about which a prediction is to be made and information given about the other parameter(s) on the basis of which this prediction is to be made. Well shortly be looking at all this. WHAT ARE QUANTITATIVE ARGUMENTS Before moving forward, lets first define the term we used: Quantitative Predictive Arguments - It basically refers to those questions that talk in terms of numbers - absolute numbers, percentages, or proportions - and make a prediction about one parameter on the basis of value of some other parameters. Hence to summarize: These questions talk about a variable, say Z, which is a function of X and Y, Z = f(X,Y) and the question (the passage or the stem) makes conclusions or predictions about Z on the basis of value of one of the parameters, say, X. So, it could say that since X is increasing, it means Z is also increasing. A simple example of this could be an argument shown below: Company A has cut down its expenses by 15% over the last year. This implies that company A has made more profits this year.

HOW TO APPROACH QUANTITATIVE PREDICTIVE ARGUMENTS For easy reference, I am going to use the acronym QPA, for this category of questions. In GMAT, QPA questions come across various question types such as Weaken, Strengthen, Evaluate, Flaw in the Reasoning, and Assumptions. It is possible to solve the above stated question types in 30 seconds or less by leveraging the structure of the QPA argument. To illustrate the same, we will take another look at the argument that we just presented. Company A has cut down its expenses by 15% over the last year. This implies that company A has made more profits this year. So, its talking about two things, Profits and Costs, and it concludes something about the profits on the basis of some chang e in costs. However, we know that Profits is a function of two variables: Revenues and Costs and is given by: Profits = Revenues Costs So, we know that costs have come down. But what about the other factor - revenues? Have they remained constant? Have they increased? Have they decreased? Nothing as such is mentioned. Now, this kind of argument can be used to create all kinds of questions. For example: An assumption could be that there were no adverse changes in the revenue figures compared to the last year

A weakener to the above argument could indicate that revenues of company A have declined over the last year. A strengthener would either imply that revenues remained constant or that the revenues increased. An evaluate question would have an answer such as, Whether the revenues of company A declined this year? A flaw in the reasoning could say something along the lines of - The argument fails to consider that there could be adverse changes in the parameters, other than the costs, which could affect profits

HOW TO FIND THE CORRECT OPTION CHOICE IN LESS THAN 30 SECONDS We have observed that these kinds of questions are quite prominent in GMAT. But how does our current understanding help us? The answer is that it helps us in prethinking (guessing) the answer before moving to the answer choices. So, whenever we see that a prediction (increase or decrease) about a variable is made on the basis of change in some other variable(s), we can immediately draw an equation on the paper about the relation of the two variables and see what other variables are there in the equation. Almost always, there will be one other variable and almost always, the answer choice is going to talk about the other variable, which is not considered in the passage or the question stem. We have provided 10 official questions on which you can apply the approach discussed above. Now, lets go back to our factory spinach example: We are given that the factory spinach costs about four times the cost of commonly sold. Now, we need to find out an option statement that supports the claim that the factory spinach could still be profitable. So, we draw the equation: Profit from spinach = Revenue from Spinach Cost of spinach Here, kindly note that we are talking about all these terms per unit of spinach. Now, the passage talks about high costs but it doesnt talk about the revenues or in this case price per unit of factory spinach. So, as per our understanding of QPA type questions, the correct option choice would be the one talking about price. We see that only option C fits this criteria and this is the answer choice. So, instead of analyzing every option choice and finding how it affects the conclusion, we are just looking for an option choice which talks about the price of factory spinach. This significantly brings down the time required to go through the options and improves accuracy since we dont get trapped in wrong but attractive answer choices. Here, its important to pay attention to the below two points: Option C is the correct choice, not only because it is talking about prices but because it is talking about prices in a manner that strengthens the given argument, which is our requirement. If this option statement had stated something about prices which does not impact the argument or weaken it, it would not have been the correct answer choice. Option E also mentions high vegetable prices but as one can easily see, this statement is primarily talking about other hydroponic facility and is thus irrelevant to the argument at hand. So, while this framework will help you eliminate irrelevant option fast, you need to evaluate the option statements remaining after elimination to see whether they fit the requirement. Lets look at the application of this understanding on a supply demand scenario. Consider this question from OG: In the past the country of Malvernia has relied heavily on imported oil. Malvernia recently implemented a program to convert heating systems from oil to natural gas. Malvernia currently produces more natural gas each year than it uses, and oil production in Malvernian oil fields is increasing at a steady pace. If these trends in fuel production and usage continue, therefore, Malvernian reliance on foreign sources for fuel is likely to decline soon. Which of the following would it be most useful to establish in evaluating the argument? (A) When, if ever, will production of oil in Malvernia outstrip production of natural gas? (B) Is Malvernia among the countries that rely most on imported oil? (C) What proportion of Malvernia's total energy needs is met by hydroelectric, solar, and nuclear power? (D) Is the amount of oil used each year in Malvernia for generating electricity and fuel for transportation increasing? (E) Have any existing oil-burning heating systems in Malvernia already been converted to natural-gas-burning heating systems?

The question is talking about import of fuel to meet the deficit in demand and supply. In equation form, it is: Import = Demand Supply

Import of Natural Gas and Oil = (Demand Supply)NG + (Demand Supply)oil Now, we know that the first parenthesized expression is negative and in the second expression, we are only told about supply of oil, not the demand, so the answer choice must be talking about demand of oil. We see that option D and option E talk about the demand of oil. So, options A, B and C can be eliminated right away. While option D directly asks whether oil consumption is increasing or not, option E talks about shifting of consumption from oil to natural gas. Since, we are concerned about import of both oil and natural gas; a shift from o il to natural gas will not impact us. Itll only change whether we need to import oil or natural gas. Thus, option D is the correct choice. So, again we can see that approaching options with an objective in mind helps us in eliminating them at a much faster speed than otherwise and time saved is score improved in GMAT. Now, lets just formalize our thinking process to apply in QPA type questions:

Here, it is important to note that the second step i.e. writing down the equation is very helpful and is a must in the beginning, since there could be times when the question will try to trick you into believing that it has talked about both the parameters even when it has not. Having an equation in the written form will help you maintain clarity. As you practice the exercise questions given along this article, youll be able to appreciate this better. HOW TO DETERMINE IF A QUESTION FALLS IN THIS CATEGORY Our question category is QPA i.e. Quantitative Predictive Arguments type. So, the question has to make some quantitative predictions (i.e. increase or decrease) in some parameter to qualify as this question type. For example, the below OG question talks about profits but its not a QPA type since there are no predictions made. Suncorp, a new corporation with limited funds, has been clearing large sections of the tropical Amazon forest for cattle ranching. This practice continues even though greater profits can be made from rubber tapping, which does not destroy the forest, than from cattle ranching, which does destroy the forest. Which of the following, if true, most helps to explain why Suncorp has been pursuing the less profitable of the two economic activities mentioned above?

(A) The soil of the Amazon forest is very rich in nutrients that are important in the development of grazing lands. (B) Cattle-ranching operations that are located in tropical climates are more profitable than cattle-ranching operations that are located in cold-weather climates. (C) In certain districts, profits made from cattle ranching are more heavily taxed than profits made from any other industry. (D) Some of the cattle that are raised on land cleared in the Amazon are killed by wildcats. (E) The amount of money required to begin a rubber-tapping operation is twice as high as the amount needed to begin a cattle ranch. EXCEPTIONS TO THE APPROACH While we have not come across an exception to our approach in the official questions researched, we cannot eliminate a scenario where an exception may arise. The scenario is when the passage talks about one of the factors affecting the outcome and the option statement also talks about that factor. For example, in our spinach factory, a correct option statement could also have been: Factory spinach has much longer shelf life than Californian spinach, a large proportion of which gets rotten during storage and transportation. So this statement has an impact on costs (storage costs) but it talks about costs after the production. So it says that while production costs may be higher for factory spinach, overall costs might be less or equal. However, this kind of scenario would constitute an exception, rather than a rule. Therefore, we would recommend that one follow the framework suggested and only if no option statement satisfies the requirement, then consider this scenario.

TAKE AWAY Draw an equation on paper which explicitly states the relationship between the parameters. Eliminate the option statements that dont talk about the other parameter in the equation. Evaluate the remaining option statements to find the correct answer.

EXERCISE PROBLEMS We have included 10 OG questions which belong to this category. Put your answers in the responses. Also include the relevant equation used in solving the question. 1. The cotton farms of Country Q became so productive that the market could not absorb all that they produced. Consequently, cotton prices fell. The government tried to boost cotton prices by offering farmers who took 25 percent of their cotton acreage out of production direct support payments up to a specified maximum per farm. The governments program, if successful, will not be a net burden on the budget. Which of the following, if true, is the best basis for an explanation of how this could be so? A. Depressed cotton prices meant operating losses for cotton farms, and the government lost revenue from taxes on farm profits. B. Cotton production in several counties other than Q declined slightly the year that the support-payment program went into effect in Q. C. The first year that the support-payment program was in effect, cotton acreage in Q was 5% below its level in the base year for the program. D. The specified maximum per farm meant that for very large cotton farms the support payments were less per acre for those acres that were withdrawn from production than they were for smaller farms. E. Farmers who wished to qualify for support payments could not use the cotton acreage that was withdrawn from production to grow any other crop. 2. In Gandania, where the government has a monopoly on tobacco sales, the incidence of smoking-related health problems has risen steadily for the last twenty years. The health secretary recently proposed a series of laws aimed at curtailing tobacco use in Gandania. Profits from tobacco sales, however, account for ten percent of Gandanias annual revenues. Therefore, Gandania cannot afford to institute the proposed l aws. Which of the following, if true, most seriously weakens the argument? A. All health care in Gandania is government-funded. B. Implementing the proposed laws is not likely to cause a significant increase in the amount of tobacco Gandania exports. C. The percentage of revenue Gandania receives from tobacco sales has remained steady in recent years. D. Profits from tobacco sales far surpass any other single source of revenue for the Gandanian government. E. No government official in Gandania has ever previously proposed laws aimed at curtailing tobacco use. 3. Printwells Ink Jet Division manufactures ink-jet printers and the ink cartridges they use. Sales of its ink-jet printers have increased. Monthly revenues from those sales, however, have not increased, because competition has forced Printwell to cut the prices of its printers. Unfortunately, Printwell has been unable to bring down the cost of manufacturing a printer. Thus, despite the increase in printer sales, the Ink Jet Division must be providing the company with much smaller than it used to. Which of the following, if true, most seriously weakens the argument? A. Ink-jet printers in regular use frequently need new ink cartridges, and Printwells printers only accept Printwells ink cartridg es. B. Unlike some competing companies, Printwell sells all of its printers through retailers, and these retailers costs account for a sizable proportion of the printers ultimate retail price. C. Some printer manufacturers have been forced to reduce the sale price of their ink-jet printers even more than Printwell has. D. In the past year, no competing manufacturer of ink-jet printers has had as great an increase in unit sales of printers as Printwell has. E. In the past year, sales of Printwells ink-jet printers have increased more than sales of any other type of printer made by Printwell. 4. In Teruvia, the quantity of rice produced per year is currently just large enough to satisfy domestic demand. Teruvias total rice acreage will not be expanded in the foreseeable future, nor will rice yields per acre increase appreciably. Teruvias population, however, will be increasing significantly for years to come. Clearly, therefore, Teruvia will soon have to begin importing rice. Which of the following is an assumption on which the argument depends? A. No pronounced trend of decreasing per capita demand for rice is imminent in Teruvia. B. Not all of the acreage in Teruvia currently planted with rice is well suited to the cultivation of rice. C. None of the strains of rice grown in Teruvia are exceptionally high-yielding. D. There are no populated regions in Teruvia in which the population will not increase. E. There are no major crops other than rice for which domestic production and domestic demand are currently in balance in Teruvia. 5. For similar cars and comparable drivers, automobile insurance for collision damage has always cost more in Greatport than in Fairmont. Police studies, however, show that cars owned by Greatport residents are, on average, slightly less likely to be involved in a collision than cars in Fairmont. Clearly, therefore, insurance companies are making a greater profit on collision damage insurance in Greatport than in Fairmont. In evaluating the argument, it would be most useful to compare A. the level of traffic congestion in Greatport with the level of traffic congestion in Fairmont B. the cost of repairing collision damage in Greatport with the cost of repairing collision damage in Fairmont C. the rates Greatport residents pay for other forms of insurance with the rates paid for similar insurance by residents of Fairmont D. the condition of Greatport's roads and streets with the condition of Fairmont's roads and streets E. the cost of collision-damage insurance in Greatport and Fairmont with that in other cities 6. Contrary to earlier predictions, demand for sugarcane has not increased in recent years. Yet, even though prices and production amounts have also been stable during the last three years, sugarcane growers last year increased their profits by more than 10% over their previous year's level. Any of the following statements, if true about last year, helps to explain the rise in profits EXCEPT: A. Many countries that are large consumers of sugarcane increased their production of sugarcane-based ethanol. yet their overall consumption decreased. B. Sugarcane growers have saved money on wages by switching from paying laborers an hourly wage to paying them by the amount harvested. C. The price of the oil, the major energy source used by sugarcane growers in harvesting their crops, dropped by more than 20%.

D. Many small sugarcane growers joined together to form an association of sugarcane producers and began to buy supplies at low group rates. E. Rainfall in sugarcane-growing regions was higher than it had been during the previous year, allowing the growers to save money on expensive artificial irrigation. 7. Which of the following most logically completes the argument below? Davison River farmers are currently deciding between planting winter wheat this fall or spring wheat next spring. Winter wheat and spring wheat are usually about equally profitable. Because of new government restrictions on the use of Davison River water for irrigation, per acre yields for winter wheat, though not for spring wheat, would be much lower than average. Therefore, planting spring wheat will be more profitable than planting winter wheat, since_______. A. the smaller-than-average size of a winter wheat harvest this year would not be compensated for by higher winter wheat prices B. new crops of spring wheat must be planted earlier than the time at which standing crops of winter wheat are ready to be harvested C. the spring wheat that farmers in the Davison River region plant is well adapted to the soil of the region D. spring wheat has uses that are different from those of winter wheat E. planting spring wheat is more profitable than planting certain other crops, such as rye 8. Insurance Company X is considering issuing a new policy to cover services required by elderly people who suffer from diseases that afflict the elderly. Premiums for the policy must be low enough to attract customers. Therefore, Company X is concerned that the income from the policies would not be sufficient to pay for the claims that would be made. Which of the following strategies would be most likely to minimize Company X's losses on the policies? A. Attracting middle-aged customers unlikely to submit claims for benefits for many years B. Insuring only those individuals who did not suffer any serious diseases as children C. Including a greater number of services in the policy than are included in other policies of lower cost D. Insuring only those individuals who were rejected by other companies for similar policies E. Insuring only those individuals who are wealthy enough to pay for the medical services 9. In the United States, of the people who moved from one state to another when they retired, the percentage who retired to Florida has decreased by three percentage points over the past ten years. Since many local businesses in Florida cater to retirees, these declines are likely to have a noticeably negative economic effect on these businesses and therefore on the economy of Florida. Which of the following, if true, most seriously weakens the argument given? A. People who moved from one state to another when they retired moved a greater distance, on average, last year than such people did ten years ago. B. People were more likely to retire to North Carolina from another state last year than people were ten years ago. C. The number of people who moved from one state to another when they retired has increased significantly over the past ten years. D. The number of people who left Florida when they retired to live in another state was greater last year than it was ten years ago. E. Florida attracts more people who move from one state to another when they retire than does any other state. 10. A discount retailer of basic household necessities employs thousands of people and pays most of them at the minimum wage rate. Yet following a federally mandated increase of the minimum wage rate that increased the retailers operating costs considerably, the retailers profits increase d markedly. Which of the following, if true, most helps to resolve the apparent paradox? A. Over half of the retailers operating costs consist of payroll expenditures; yet only a small percentage of those expenditures go to pay management salaries. B. The retailers customer base is made up primarily of people who earn, or who depend on the earnings of others who earn, th e minimum wage. C. The retailers operating costs, other than wages, increased substantially after the increase in the minimum wage rate went into effect. D. When the increase in the minimum wage rate went into effect, the retailer also raised the age rate for employees who had been earning just above minimum wage. E. The majority of the retailers employees work as cashiers, and most cashiers are paid the minimum wage.
_________________

Вам также может понравиться